econ 360 test questions

Ace your homework & exams now with Quizwiz!

Suppose that the U.S. Open ticket costs $100 and the British Open ticket costs £50 and the exchange rate is $1.43. How much does the British Open ticket cost for an American attending the British Open?

$1.43 * 50 = $71.50

Which type of exchange rate system minimizes external shocks to an economy?

A flexible exchange rate system

Given that Sandy can produce 10 economics reports or 2 sales calls and Tim can produce 2 economics reports or 1 sales call, which of the following would NOT be a mutually agreeable terms of trade for Sandy and Tim? A) 1 economics report for 1 sales call B) 1 sales call for 3 economics reports C) 1 sales call for 4 economics reports D) 1 economics report for 1/4 of a sales call

A) 1 economics report for 1 sales call

If two countries agree to specialize and trade based on comparative advantage, which of the following is most likely to be TRUE? A) Both of the countries will consume outside their respective production possibilities curves. B) One of the countries will end up receiving all of the gains from trade. C) One of the countries will both consume and produce on its production possibilities curve. D) Only one of the countries will produce on and consume outside its production possibilities curve.

A) Both of the countries will consume outside their respective production possibilities curves.

Which of the following is FALSE? A) Consumer surplus increases after a tariff is placed on imports. B) Producer surplus increases after a tariff is imposed. C) Government revenue increases after a tariff is imposed. D) Deadweight losses result from tariffs.

A) Consumer surplus increases after a tariff is placed on imports.

Which of the following is NOT a reason social returns might be greater than private returns? A) Excess competition between firms B) Knowledge spillovers C) Spillovers from research and development D) Capital market imperfections

A) Excess competition between firms

Which of the following is NOT one of the major problems with expanding the EU? A) Expansion has become a more difficult task because of the unwillingness of the eastern and central European countries to change. B) The programs that target EU expenditures could be stretched thin by the addition of countries with much lower incomes. C) The governance structure of the EU had to be changed to avoid becoming unwieldy and ineffective. D) The EU may be faced with an unstable eastern border with huge worker migratory flows if the transition economies fail. E) Most central and eastern European countries have large agricultural sectors and extending subsidies to these countries would entail an enormous flow of funds given the Common Agricultural Policy.

A) Expansion has become a more difficult task because of the unwillingness of the eastern and central European countries to change.

Which of the following is NOT an expected benefit of reducing nontariff barriers to trade? A) Fewer firms to compete with B) Lower prices for many goods C) Increase in the volume of exports and imports D) Increase in production levels E) Improved overall economic welfare

A) Fewer firms to compete with

Which of the following is a macroeconomic factor that contributed to the financial crisis in 2007? A) Global saving and investment imbalances B) Financial market innovation C) Deeper levels of integration across financial markets D) Challenges and failures in financial regulation

A) Global saving and investment imbalances

Which of the following is most important in determining the number of jobs in an economy? A) Macroeconomic policies B) Trade policies C) Urban policies D) Environmental policies E) Tax policies

A) Macroeconomic policies

Suppose that Canada can produce 15 units of timber or 3 units of grain. Suppose that Mexico can produce 6 units of timber or 2 units of grain. Which of the following is CORRECT? A) Mexico has a comparative advantage in grain production. B) Mexico has an absolute advantage in timber production. C) Canada has a comparative advantage in grain production. D) The countries would find trade mutually beneficial at a trading ratio of 1 grain for 2 timber.

A) Mexico has a comparative advantage in grain production.

Suppose Paraguay can produce 12 wheat or 3 corn. Suppose Bolivia can produce 4 wheat or 2 corn. Suppose opportunity costs are constant. Given these production possibilities, A) Paraguay has a comparative advantage in wheat. B) Paraguay has a comparative advantage in corn. C) Paraguay has a comparative advantage in both goods. D) Paraguay has an absolute advantage in neither good.

A) Paraguay has a comparative advantage in wheat.

According to the principle of subsidiarity, in which of the following areas should the authority for making decisions be taken from national governments and given to the EU? A) Pollution of a trans-national waterway B) Limits on the hours retails stores are allowed to be open C) Highway traffic laws D) Recycling requirements on packaging E) All of the above.

A) Pollution of a trans-national waterway

Which type of restriction on quantity of imports is the most transparent? A) Quota B) Licensing requirements C) Voluntary export restraints D) Government procurement policies

A) Quota

Given that Sandy can produce 10 economics reports or 2 sales calls and Tim can produce 2 economics reports or 1 sales call, which of the following is FALSE? A) Sandy has a comparative advantage in sales calls. B) Tim has a comparative advantage in sales calls. C) Sandy has a comparative advantage in economics reports. D) Sandy has an absolute advantage in both economics reports and sales calls.

A) Sandy has a comparative advantage in sales calls.

Given that Sandy can produce 10 economics reports or make 2 sales calls and Tim can produce 2 economics reports or make 1 sales call, we can conclude that A) Sandy should specialize in economics reports, and Tim should specialize in sales calls. B) Sandy should produce both economics reports and sales calls since she cannot possibly gain from trade with Tim. C) Tim should specialize in producing economics reports, and Sandy should specialize in producing sales calls. D) Tim should produce both economics reports and sales calls.

A) Sandy should specialize in economics reports, and Tim should specialize in sales calls.

Suppose that Sandy can produce 10 economic reports or make 2 sales calls. Suppose Tim can produce 2 economic reports or make 1 sales call. Which of the following is CORRECT? A) The opportunity cost for Sandy of producing one economics report is 1/5 of a sales call. B) The opportunity cost for Sandy of producing one sales call is 10 economics reports. C) The opportunity cost for Tim of producing one sales call is 1/2 of an economics report. D) The opportunity cost for Tim of producing one economics report is 2 sales calls.

A) The opportunity cost for Sandy of producing one economics report is 1/5 of a sales call.

Which of the following treaties/agreements would be associated with creating a free trade area for the European Union? A) Treaty of Rome B) Single Europe Treaty C) Delors Agreement D) Treaty on European Union E) Schengen Agreement

A) Treaty of Rome

After trade opens, the short run impact on the income of the specific factor that is relatively scarce will be A) a decrease in its income. B) an increase in its income. C) no change in its income. D) indeterminate, income effects are not possible to know.

A) a decrease in its income.

An exchange rate crisis is caused by A) a sudden and an unexpected collapse in the value of a nation's currency. B) the inability of the IMF to predict the immediate collapse of the currency of a country. C) the adoption of a flexible exchange rate system by a country or group of countries. D) the adoption of a fixed exchange rate system by a country or group of countries. E) Both C and D are correct.

A) a sudden and an unexpected collapse in the value of a nation's currency.

Suppose that the nominal exchange rate between the U.S. dollar and the Canadian dollar is 0.75 U.S. dollars per Canadian dollar. If Canada's rate of inflation is 0 percent and the U.S. rate is 10 percent, then the real exchange rate for the U.S. dollar will A) appreciate by about 9 percent. B) appreciate by 10 percent. C) depreciate by about 9 percent. D) depreciate by 10 percent. E) None of the above.

A) appreciate by about 9 percent.

Suppose that Brazil is capital abundant and Chile is natural resource abundant. If timber is natural resource intensive and computers are capital intensive, then according to the Stolper-Samuelson Theorem, the incomes of the owners of ________ are likely to rise in Brazil after trade with Chile begins. A) capital B) labor C) natural resources D) It is impossible to determine which will be favored.

A) capital

With the full implementation of the Single European Act, the EU became a A) common market. B) free trade area. C) customs union. D) economic union. E) None of the above.

A) common market.

A production possibilities curve that is a straight line represents the case of A) constant costs. B) increasing costs. C) decreasing costs. D) constant opportunity costs but increasing real costs. E) constant opportunity costs but decreasing real costs.

A) constant costs.

Tariffs reallocate income from A) consumers to producers. B) producers to consumers. C) government to producers. D) consumers to foreigners. E) Both A and D

A) consumers to producers.

Changes in aggregate demand A) could be caused by changes in the spending decisions of the households, businesses, the government, and foreigners. B) are very uncommon. C) are unlikely to change quickly in response to economic events. D) are primarily based on changes in firms' abilities to produce products. E) are not affected by changes in government policies.

A) could be caused by changes in the spending decisions of the households, businesses, the government, and foreigners.

All of the following are possible outcomes of a banking crisis except A) depositors, but not banks, may lose all or a portion of their assets. B) a recession due to decreases in consumption by households. C) decreases in lending practices by banks. D) decreases in investment. E) a contagion effect of the crisis from vulnerable banks to financial institutions on sound basis.

A) depositors, but not banks, may lose all or a portion of their assets.

Suppose the dollar is subject to a floating exchange rate system and that R is the number of dollars per unit of foreign exchange. If R increases, then the dollar A) depreciates. B) appreciates. C) is devalued. D) is revalued. E) Both A and C.

A) depreciates.

Intraindustry trade relies on A) economies of scale. B) the product cycle. C) differences in factor endowments. D) government industrial policies. E) monopoly pricing.

A) economies of scale.

A real cost of tariffs and quotas that is difficult to measure is that they A) encourage rent seeking. B) shift income from consumers to producers. C) limit the quantity of imports. D) reduce wages. E) cause deflation.

A) encourage rent seeking.

The single most important reason for Canada's seeking a free trade agreement with the United States was to A) ensure its access to the U.S. market. B) ensure its ability to join NAFTA. C) harmonize its environmental laws with the United States. D) avoid international outsourcing in low wage countries. E) ensure the continuation of its social programs.

A) ensure its access to the U.S. market.

When spending and incomes in an economy increase, A) imports are likely to increase. B) imports are likely to be unchanged. C) imports are likely to decrease. D) exports are likely to decrease.

A) imports are likely to increase.

Based on Table 4.1, according to the Heckscher-Ohlin Theorem, U.S. exports should be goods that A) intensively use labor input. B) intensively use capital input. C) use capital and labor in about equal proportions. D) use either labor or capital input, depending on the good. E) Not enough information to tell.

A) intensively use labor input.

Intraindustry trade refers to A) international trade of products made within the same industry. B) international trade of products made across different industries. C) trade that occurs as a result of comparative advantage. D) the exchange of non-similar items. E) trade that occurs mostly within developing countries.

A) international trade of products made within the same industry.

The straight-line production possibilities curve introduced in the text A) is not subject to increasing opportunity costs. B) fails to reflect tradeoffs. C) fails to benefit trading nations. D) refutes the principles of comparative advantage. E) All of the above.

A) is not subject to increasing opportunity costs.

President Salinas of Mexico devised a strategy to restore Mexican growth by encouraging A) large inflows of foreign capital. B) large increases in domestic savings. C) large interest rate cuts. D) an expansion of import substitution industrialization policies. E) more government ownership of industrial firms.

A) large inflows of foreign capital.

A lender of last resort A) makes loans when no one else will. B) makes loans without regard for risk. C) is a firm that is forced to make loans for its own survival. D) Both A and B. E) None of the above.

A) makes loans when no one else will.

A single currency area requires A) mobile labor and synchronized business cycles. B) immobile labor and synchronized business cycles. C) immobile labor and mobile capital. D) a political union. E) mobile labor and unsynchronized business cycles.

A) mobile labor and synchronized business cycles.

The main policy advice given by the IMF to East Asian countries facing the financial crises of 1997/1998 was A) raising their domestic interest rates to stabilize the collapsing currencies. B) using their monetary and fiscal policies alone. C) use capital controls. D) adopting a flexible exchange rate system. E) adopting a fixed exchange rate system.

A) raising their domestic interest rates to stabilize the collapsing currencies.

Suppose Mexico can produce 5 autos or 10 corn. Suppose the United States can produce 4 autos or 20 corn. If opportunity costs are constant for both countries, then A) the United States has a comparative advantage in corn production. B) Mexico has a comparative advantage in corn production. C) the United States cannot gain from trade with Mexico. D) the United States has a comparative advantage in auto production.

A) the United States has a comparative advantage in corn production.

Under a fixed exchange standard, if the domestic demand for foreign exchange increases A) the central monetary authority must meet the demand out of its reserves. B) the central monetary authority must increase the supply of domestic money. C) the fixed exchange standard will breakdown. D) inflation will increase. E) the domestic currency must be depreciated.

A) the central monetary authority must meet the demand out of its reserves.

If the trade line that passes through the production point on the PPC has a slope that is shallower than the slope of the PPC at the same point, then A) the country can get greater gains from trade if it moves production away from the vertical axis. B) the country can get greater gains from trade if it moves production toward the vertical axis. C) the country cannot improve on its gains from trade. D) There are no gains from trade in this example. E) There is not enough information to tell how it can improve its gains from trade.

A) the country can get greater gains from trade if it moves production away from the vertical axis.

The European Union membership criteria includes all of the following except A) the country must participate in free trade with all of its goods and services. B) the country must follow market-based economics. C) the country must formally adopt the EU-wide rules such as technical standards, environmental inspections, banking supervision, etc. D) the country must be a stable functioning democracy. E) None of the above.

A) the country must participate in free trade with all of its goods and services.

Suppose that a country is producing on its PPC at a point to the left of the tangency between the trade line and the PPC. At the production point, A) the opportunity cost in production of the good on the horizontal axis is less than its trade price. B) the opportunity cost in production of the good on the horizontal axis is more than its trade price. C) the opportunity cost in production of the good on the vertical axis is less than its trade price. D) the opportunity cost in production of the good on the horizontal axis may be either less than or more than its trade price. E) the opportunity cost in production of the good on the vertical axis equals its trade price.

A) the opportunity cost in production of the good on the horizontal axis is less than its trade price.

A country possesses a comparative advantage in the production of a product if A) the opportunity cost, in terms of the amount of other products that it gives up to produce this product, is lower than it is for its trading partners. B) it possesses an absolute advantage in the production of this good compared to its trading partners. C) it is able to produce less of this good per worker than its trading partners. D) it can produce more of this good per hour than its trading partners.

A) the opportunity cost, in terms of the amount of other products that it gives up to produce this product, is lower than it is for its trading partners.

When aggregate demand increases, A) the price level is likely to rise as GDP rises. B) the price level is likely to fall as GDP rises. C) aggregate supply will shift to the right. D) aggregate supply will shift to the left.

A) the price level is likely to rise as GDP rises.

Tijuana, Mexico is across the border from San Diego, California. It has become a world-leading producer and exporter of television sets and computer monitors, which it assembles in modern factories owned by multinational consumer electronics firms such as Sony. Initially, these electronics were produced in the industrialized countries of their parent companies, and after several years, the production moved to Tiajuana. This is an example of A) the product cycle. B) intraindustry trade. C) the specific factors model. D) the magnification effect. E) None of the above.

A) the product cycle.

Covered interest arbitrage involves both A) the purchase of a foreign asset and a forward contract in the market for foreign exchange. B) the purchase of a domestic asset and a spot contract in the market for foreign exchange. C) the sale of a foreign asset and the purchase of a forward contract in the market for foreign exchange. D) the sale of domestic stocks and the purchase of foreign bonds. E) None of the above.

A) the purchase of a foreign asset and a forward contract in the market for foreign exchange.

The first and biggest problem the EU faces in its expansion to the east is A) the reform of its agricultural subsidy programs. B) the lack of democracy in the countries that are most likely to become members. C) the unwillingness of the new members to adopt EU rules. D) the lack of market economies in the countries that are most likely to become members. E) the unwillingness of citizens in the new member countries to migrate to higher income countries.

A) the reform of its agricultural subsidy programs.

In order for large countries to successfully use tariffs to increase well being, A) they must have significant market power so that foreign firms will cut prices to preserve their sales. B) the deadweight loss created by the tariff must be greater than the government revenue the tariff generates. C) domestic production must increase more significantly than for the small country case. D) domestic consumption and imports must decrease more significantly than in the small country case.

A) they must have significant market power so that foreign firms will cut prices to preserve their sales.

Based on Table 3.1, the pre-trade relative price of a computer in Mexico is A) three pairs of shoes. B) one pair of shoes. C) one-half pair of shoes. D) one-third pair of shoes. E) None of the above.

A) three pairs of shoes.

The "Four Freedoms" do NOT include the right to A) vote in local elections. B) migrate within the EU. C) write insurance policies throughout the EU. D) open bank accounts anywhere in the EU. E) move goods from one country to another inside the EU.

A) vote in local elections.

If the world price for a good is above a nation's pre-trade equilibrium price, then the nation A) will export the good. B) will import the good. C) will neither export nor import the good. D) cannot gain from trade. E) Both C and D.

A) will export the good.

Offshoring required which types of advances?

Advances in communications technology, such as fax machines, video conferencing, the internet, and in general, lower prices for long distance communication

What are the two largest expenditure categories in the EU budget?

Agricultural support; cohesion funds

According to your text, what is the major long-run issue facing the European Union?

An aging population

If inflation is higher in the home market, what is expected to happen to the real value of the home currency as time passes?

Appreciates

Based on Scenario 6.1 above, if a tariff of 20 percent is placed on imports of dining room tables, the effective rate of protection is A) 20 percent. B) 25 percent. C) 30 percent. D) 40 percent. E) There is not enough information to tell.

B) 25 percent.

Which of the following institutions is the most important participant in foreign currency markets? A) A retail customer B) A commercial bank C) A foreign exchange broker D) A central bank E) None of the above.

B) A commercial bank

Suppose that Paraguay can produce 12 wheat or 3 corn and Bolivia can produce 4 wheat or 2 corn. Suppose that opportunity costs are constant. Which of the following is a potentially agreeable trade arrangement for Paraguay and Bolivia? A) Paraguay trades one corn to Bolivia for three units of wheat. B) Bolivia trades one corn to Paraguay for three units of wheat. C) Bolivia trades one corn to Paraguay for one wheat. D) Paraguay trades one wheat to Bolivia for two corn.

B) Bolivia trades one corn to Paraguay for three units of wheat.

Before the creation of the European Economic Community, there was the A) European Economic Union. B) European Coal and Steel Community. C) European Union. D) European Free Trade Area. E) Single European Community.

B) European Coal and Steel Community.

If a component of aggregate demand increases, A) GDP in the United States is likely to increase less than that component of spending increased. B) GDP in the United States is likely to increase more than that component of spending increased. C) GDP in the United States is likely to decrease. D) GDP in the United States will not change.

B) GDP in the United States is likely to increase more than that component of spending increased.

Which of the following does not occur in resolving a debt crisis? A) Debts are restructured B) Repayment periods are shortened C) Interest rates are reduced D) Some partial debt forgiveness

B) Repayment periods are shortened

The "Four Freedoms" are an important goal of the A) Treaty of Rome. B) Single European Act. C) European Monetary System. D) Maastricht Treaty. E) European Free Trade Area.

B) Single European Act.

Which of the following is FALSE? A) Most of the maquiladora industry is located in the states of Mexico that border the United States. B) The maquiladora industry has not created much employment in Mexico because most of the production is capital intensive. C) The maquiladora industry accounts for more than half of Mexico's exports today. D) Growth in Mexico's northern border cities , such as Tijuana and Ciudad Juarez, took off almost a decade before NAFTA because of the maquiladora industry.

B) The maquiladora industry has not created much employment in Mexico because most of the production is capital intensive.

Which of the following was NOT one of the causes of the Asian financial crises of 1997 and 1998? A) A current account deficit and financial account surpluses B) The use of exports as an engine of economic growth by the countries involved C) China's 1994 devaluation of its fixed exchange rate D) The appreciation of the U.S. dollar and depreciation of the Japanese yen E) Crony capitalism

B) The use of exports as an engine of economic growth by the countries involved

According to the text, which of the following factors may make the theory of purchasing power parity unrealistic? A) Purchasing power parity works only with traded goods. B) Trading countries may stop exchanging goods once prices between them equalize. C) Shipping, insurance, and transaction costs may reduce the implication of purchasing power parity. D) Prices may not equalize if goods arbitrage is reduced by trade barriers. E) The effects of purchasing power parity may not show up until many years have passed.

B) Trading countries may stop exchanging goods once prices between them equalize.

If one nation is able to produce a good at a lower opportunity cost than another, it has A) an absolute advantage in that good. B) a comparative advantage in that good. C) a productivity advantage in that good. D) a technological advantage in that good. E) no reason to want to trade that good.

B) a comparative advantage in that good.

The Mexican peso crisis of 1994 and 1995 was directly related to A) the start of NAFTA. B) a large capital account surplus. C) a large capital account deficit. D) an undervalued peso. E) a large current account surplus.

B) a large capital account surplus.

A flexible exchange rate system crisis involves A) a revaluation of the currency. B) a rapid and uncontrolled depreciation of the currency. C) a decrease in the dollar value of the country's international debt. D) a sure political collapse of the ruling government. E) All of the above.

B) a rapid and uncontrolled depreciation of the currency.

Expenditure switching refers to A) a switching back and forth between investment and consumption expenditures. B) a switching back and forth between domestic and foreign goods in response to changes in the exchange rate. C) a switching back and forth between domestic and foreign goods in response to changes in the interest rate. D) a switching of back and forth in the current account from a deficit to a surplus and vice versa. E) All of the above.

B) a switching back and forth between domestic and foreign goods in response to changes in the exchange rate.

Producer surplus is equal to the area A) under the demand curve and above the supply curve. B) above the supply curve and below the price line. C) under the demand curve. D) under the supply curve. E) under the demand curve and above the price line.

B) above the supply curve and below the price line.

If consumption spending increases because people feel more confident about the future, A) aggregate demand will shift to the left. B) aggregate demand will shift to the right. C) aggregate supply will shift to the left. D) aggregate supply will shift to the right.

B) aggregate demand will shift to the right.

Based on the theory of comparative advantage, nations maximize their well being when they A) create more jobs. B) allocate resources more efficiently. C) increase trade surpluses. D) increase exports.

B) allocate resources more efficiently.

An austerity policy is A) an increase in the money supply. B) an expenditure reduction and expenditure switching policy. C) an expansionary fiscal policy accompanied by decreases in taxes, increases in expenditures, or both. D) an exchange rate switching policy from a fixed to a flexible exchange rate system. E) None of the above.

B) an expenditure reduction and expenditure switching policy.

A slowing U.S. economy and increased enforcement of immigration laws will reduce ________ factors for Mexican immigration to the United States. A) demand growth B) demand pull C) supply push D) supply growth E) network

B) demand pull

Suppose that the nominal exchange rate between the U.S. dollar and the Mexican peso is 0.10 dollars per peso. If Mexico's inflation is 10 percent and the United States' inflation is 0 percent, from the U.S. point of view, the real exchange rate A) appreciates to 0.11 dollars per peso. B) depreciates to 0.11 dollars per peso. C) appreciates to 0.09 dollars per peso. D) depreciates to 0.09 dollars per peso. E) appreciates to 0.2 dollars per peso.

B) depreciates to 0.11 dollars per peso.

Under some circumstances, trade could stifle the development of new industries and reduce global efficiency. All of the following describe conditions that could lead to that situation except A) an initial head start gives a scale advantage to already existing firms in one country. B) diseconomies of scale make it impossible for new firms to enter the market. C) a location has a better-developed linkage between suppliers and producers, giving it a cost advantage. D) a historical accident, such as the shifting of airplane production to the United States to avoid World War II bombings, causes firms in one location to have a competitive advantage.

B) diseconomies of scale make it impossible for new firms to enter the market.

The currency crisis of 1992 caused France and a number of other countries to choose between A) a single currency for the EU and keeping their own currency. B) doing the right thing for their domestic economy and defending the exchange rate. C) lowering interest rates and reducing unemployment. D) competitive devaluations and falling unemployment. E) the Maastricht Treaty and the Single European Act.

B) doing the right thing for their domestic economy and defending the exchange rate.

All of the following are possible explanations for why it took so long for trade balances to respond to the depreciation of the dollar except A) the prior increase in the value of the dollar had padded the profit margins of foreign producers. B) foreign trade barriers made it impossible for the United States to substantially expand exports it bought. C) there were still impacts from earlier appreciations working through the system. D) exports began to increase from a much lower base than imports. E) None of the above. That is, A, B, C, and D are all correct.

B) foreign trade barriers made it impossible for the United States to substantially expand exports it bought.

A production possibilities curve that is bowed out represents the case of A) constant costs. B) increasing costs. C) decreasing costs. D) internal costs. E) external costs.

B) increasing costs.

In most cases, expenditure-switching policies must be accompanied by expenditure-reducing policies because A) expenditure-switching policies are completely ineffective without expenditure-reducing policies. B) inflation ensues as home country domestic expenditures switch away from foreign goods to domestic goods unless overall expenditures are reduced. C) inflation abroad may increase the demand for domestic goods, causing inflation to rise. D) the depreciation in the exchange rate may decrease the domestic price of foreign goods, causing an increase in the current account deficit. E) None of the above.

B) inflation ensues as home country domestic expenditures switch away from foreign goods to domestic goods unless overall expenditures are reduced.

Under a gold standard, countries should A) keep the supply of their domestic money constant. B) keep the supply of their domestic money fixed in proportion to their gold holdings. C) keep the supply of foreign exchange less than their domestic money supply. D) restrict the demand for foreign goods. E) outlaw speculation.

B) keep the supply of their domestic money fixed in proportion to their gold holdings.

If a nation has no absolute advantage, then it A) cannot gain from trade. B) still gains from trade. C) can only gain from trade if it raises its productivity levels. D) can only gain from trade if it reduces wages paid. E) can only gain from trade if it produces outside its production possibilities curve.

B) still gains from trade.

All of the following make the use of fiscal policy less attractive except A) expansionary fiscal policy tends to cause inflation and offsets some of the increased consumer spending. B) that it cannot be effective, unless it is accommodated with expansionary monetary policy. C) the substantial margin of error in the value of the multiplier. D) the legislative lag, which is the time it takes for Congress and the President to pass and implement the measure. E) the crowding out effect, which is the decrease in private spending that occurs due to increased government spending.

B) that it cannot be effective, unless it is accommodated with expansionary monetary policy.

One reason why most economists forecast that the effects of NAFTA on the U.S. economy would be small is because A) NAFTA does not bring down tariffs far enough. B) the Mexican economy is small relative to the U.S. economy. C) NAFTA trade opening provisions only cover a handful of sectors. D) Mexico was not expected to live up to its obligations under the agreement. E) the United States was not expected to live up to its obligations under the agreement.

B) the Mexican economy is small relative to the U.S. economy.

When economists talk about the gains from trade they mean that A) no one ever gets hurt by trade. B) the benefits of trade outweigh the losses. C) business firms benefit from trade but not necessarily individuals. D) trade increases government revenue through taxes on imports. E) economic restructuring is usually quick and painless.

B) the benefits of trade outweigh the losses.

One problem with NAFTA that is not shared by other trade blocs is that A) national environmental laws differ greatly. B) the countries are at very different stages of economic development. C) the countries speak different languages. D) the countries are not natural trading partners. E) the United States and Mexico have a history of conflict.

B) the countries are at very different stages of economic development.

In a small country, the net national cost of tariff protection is equal to the reduction in consumer surplus minus A) the gain to foreigners. B) the increase in government revenue and the increase in producer surplus. C) the increase in government revenue. D) the increase in producer surplus. E) the efficiency loss and the consumption side loss

B) the increase in government revenue and the increase in producer surplus.

For a country in autarky, the opportunity cost of the good on the horizontal axis is the same as A) the relative price of the good on the vertical axis. B) the relative price of the good on the horizontal axis. C) the opportunity cost of the good on the vertical axis. D) Both A and C. E) None of the above.

B) the relative price of the good on the horizontal axis.

The J-curve effect of a currency depreciation results is due to A) the initial effect having positive effects on the current account balance. B) the value of imports increasing by more than the value of exports at the time of devaluation. C) exports and imports being totally unresponsive to changes in exchange rates. D) decreases in the dollar price of imports. E) None of the above.

B) the value of imports increasing by more than the value of exports at the time of devaluation.

Based on Table 3.1, if the world price of computers is four pairs of shoes, then the optimal strategy for each country would be A) to specialize in shoes. B) to specialize in computers. C) for Mexico to specialize in shoes, and the United States in computers. D) for the United States to specialize in shoes, and Mexico in computers. E) None of the above.

B) to specialize in computers.

If social returns to the production of a good are greater than private returns, then we can conclude that relative to the social optimum, the good will be A) over produced and under priced. B) under produced and over priced. C) over produced and over priced. D) under produced and under priced. E) None of the above.

B) under produced and over priced.

In our simple trade model, having a comparative advantage in a product implies that a country will specialize completely in the product A) with the highest opportunity cost. B) with the lowest opportunity cost. C) where total output is lower per worker-hour. D) where total output is greater per worker-hour.

B) with the lowest opportunity cost.

Based on Scenario 6.1 above, value added in the United States is A) $500. B) $600. C) $400. D) $300. E) None of the above.

C) $400.

Suppose a manufacturer of software develops a new computer program that sells for $50. The $50 cost includes $0.25 for the CD it is stored on, $5 for the labor of the company software programmers, and $1.75 for packaging materials and transportation costs. Value added by the software company is A) $49.75. B) $48.25. C) $48. D) $44.75. E) $43.

C) $48.

Which of the following is TRUE? A) Points along the aggregate supply curve show the equilibrium levels of output and prices that are consistent on the demand side of the economy. B) Points along the aggregate demand curve show the equilibrium levels of output and prices that are consistent on the supply side of the economy. C) Aggregate demand shows the levels of GDP and prices where expenditure decisions and production decisions match. D) Aggregate supply shows the levels of GDP and prices where expenditure decisions and production decisions match.

C) Aggregate demand shows the levels of GDP and prices where expenditure decisions and production decisions match.

Suppose that Brazil is capital abundant and Chile is natural resource abundant. If timber is natural resource intensive and computers are capital intensive, then A) Chile will produce more computers after trade begins with Brazil. B) Brazil will produce more timber after trade begins with Chile. C) Chile will produce more timber after trade begins with Brazil. D) Brazil will completely specialize in computers once trade begins with Chile.

C) Chile will produce more timber after trade begins with Brazil.

Comparative advantage has mixed results when it comes to predicting a country's trade patterns. Which of the following is FALSE? A) There are many potential products an economy might export that use the same comparative advantage. B) A large share of international trade is not based on comparative advantage. C) Comparative advantage has proven completely incapable of predicting trade. D) Comparative advantage is a dynamic concept, which means that the spread of technology, improvement in skills, and learning-by-doing may alter a country's comparative advantage over time.

C) Comparative advantage has proven completely incapable of predicting trade.

The international institution that serves as a lender of last resort is called the A) IBRD. B) WTO. C) IMF. D) World Bank. E) GATT.

C) IMF.

Which of the following is TRUE about monopolistic competition? A) One firm serves as the entire industry. B) A small number of firms serve the entire market. C) It is competition among many firms producing similar but differentiated products. D) The pattern of production and trade is difficult to predict. E) It enjoys no economies of scale.

C) It is competition among many firms producing similar but differentiated products.

Which of the following is FALSE about intraindustry trade? A) Economies of scale allow firms to enjoy lower average costs. B) It creates gains from trade. C) It is due to comparative advantage. D) It may involve heightened competition and lower prices for consumers. E) It increases consumer choice.

C) It is due to comparative advantage.

Which one of the following countries refused to accept the IMF conditions during the East Asian financial crisis? A) South Korea B) Indonesia C) Malaysia D) Thailand E) Singapore

C) Malaysia

Which of the following is NOT a reason Mexico and Canada wanted a free trade agreement with the United States? A) Mexico wanted increased direct foreign investment. B) Canada wanted to guarantee access to the U.S. market. C) Mexico wanted the United States to lower its high barriers to Mexican products. D) Canada wanted its firms to become more globally competitive. E) Mexico wanted to institutionalize its economic reforms.

C) Mexico wanted the United States to lower its high barriers to Mexican products.

Which of the following may NOT serve as a possible chain reaction for either fiscal or monetary policy? A) G↑ ⇒ Y↑ ⇒ C↑ ⇒ Y↑ ⇒ C↑.... B) T↓ ⇒ Y↑ ⇒ C↑ ⇒ Y↑ ⇒ C↑.... C) M↑ ⇒ i↓ ⇒ I↓ ⇒ Y↓ ⇒ C↓.... D) M↑ ⇒ i↓ ⇒ I↑ ⇒ Y↑ ⇒ C↑.... E) M↓ ⇒ i↑ ⇒ I↓ ⇒ Y↓ ⇒ C↓....

C) M↑ ⇒ i↓ ⇒ I↓ ⇒ Y↓ ⇒ C↓....

Which of the following nations DOES use the euro and participates in the Treaty on European Union? A) Sweden B) Denmark C) Portugal D) The United Kingdom E) Norway

C) Portugal

Which of the following is the treaty that took the participating countries from a free trade area to a common market? A) The Treaty of Rome B) The Maastricht Treaty C) Single European Act D) Treaty on European Union

C) Single European Act

Which of the following would be associated with the early phase of the product cycle? A) Large amounts of production in low-income, developing countries B) A standardized product with an assembly-line style production process C) Sophisticated marketing and customer feedback mechanisms D) More consumption in low-income, developing countries

C) Sophisticated marketing and customer feedback mechanisms

Which of the following is TRUE according to the case study on U.S. / China trade presented in the chapter? A) China has relative abundance in capital. B) China has relative abundance in skilled labor. C) The United States has relative scarcity in unskilled labor. D) The United States has relative scarcity in capital.

C) The United States has relative scarcity in unskilled labor.

Certain kinds of tropical fruits are impossible to grow outdoors in the United States. Suppose, however, that in order to create jobs in Wyoming, the U.S. government offered extensive subsidies to firms to produce bananas. With the subsidies, firms could build greenhouses and offer the fruit at world prices. A) The United States now has a comparative advantage in bananas. B) The United States has a comparative advantage, but is not competitive. C) The United States is competitive, but does not have a comparative advantage. D) The United States has a comparative advantage and is competitive. E) None of the above.

C) The United States is competitive, but does not have a comparative advantage.

Which of the following was NOT a cause or a characteristic of the 1994/95 Mexican peso crisis? A) An overvalued exchange rate B) An inflow of large foreign portfolio capital C) The inability of the IMF, the world bank, and the NAFTA member countries (i.e., the United States and Canada) to predict the looming financial crisis D) Shifts by the world capital markets toward more conservative and risk-averse investments because of interest and exchange rate movements around the world E) High domestic investments with insufficient domestic savings

C) The inability of the IMF, the world bank, and the NAFTA member countries (i.e., the United States and Canada) to predict the looming financial crisis

Using the specific factors model, assume that strawberry production requires the specific factor of land, tractor production requires the specific factor of capital, and labor is variable. If the United States is capital abundant compared to Mexico, and Mexico is land abundant compared to the United States, then in the short run with trade, which of the following is TRUE? A) Mexican wages will rise more than the increase in the price of tractors in Mexico. B) U.S. wages will rise less than the fall in the price of tractors in the United States. C) The owners of capital in the United States will see a larger increase in their incomes in percentage terms than the increase in the price of tractors. D) The owners of land in Mexico will see a smaller increase in their incomes in percentage terms than the increase in the price of strawberries.

C) The owners of capital in the United States will see a larger increase in their incomes in percentage terms than the increase in the price of tractors.

Which of the following is a FALSE statement concerning purchasing power parity? A) Purchasing power parity states that dollars will tend to exchange for pounds at a rate that maintains a constant purchasing power of a given quantity of a currency. B) Over the long term, a Big Mac in New York will tend to cost the same as a Big Mac in London. C) There should not be significant deviations in the long-run value of purchasing power parity. D) Over the long run, purchasing power parity exerts influence over exchange rates. E) An overvalued dollar buys more in Britain than it does in the United States.

C) There should not be significant deviations in the long-run value of purchasing power parity.

Which of the following is an example of intraindustry trade? A) Trading peanut oil for tractors B) Trading crude oil for automobiles C) Trading Nokia smartphone for Apple iPhones D) Trading jeans for cotton

C) Trading Nokia smartphone for Apple iPhones

The founding document of the European Economic Community, and the document that continues to provide the basis for the European Union is the A) Treaty of Brussels. B) Delors Report. C) Treaty of Rome. D) Single European Treaty. E) Maastricht Treaty.

C) Treaty of Rome.

Which of the following is NOT a situation providing a potential advantage for Mexico that makes it competitive compared to China in trade with the United States? A) The product line requires quick turn-around time from or to delivery. B) The need to manage a just-in-time inventory system C) Wages are low and the production process is labor intensive. D) The product is heavy and bulky relative to its final value.

C) Wages are low and the production process is labor intensive.

A narrow target zone exchange rate band (such as the EEC had until 1992) is most similar to A) a flexible exchange rate system. B) a single currency. C) a fixed exchange rate system. D) an undervalued currency. E) a managed floating exchange rate.

C) a fixed exchange rate system.

Economic growth would be illustrated by A) a rightward shift of aggregate demand. B) a leftward shift of aggregate demand. C) a rightward shift of aggregate supply. D) a leftward shift of aggregate supply.

C) a rightward shift of aggregate supply.

Twenty-four years before CUSTA, another agreement between the same countries covered trade in A) textiles. B) steel. C) autos. D) agriculture. E) telecommunications.

C) autos.

Using the HO model, assume that the United States is capital abundant and Mexico is labor abundant. If soybeans are capital intensive and avocados are labor intensive, A) Mexico will produce more soybeans once trade is introduced. B) the United States will produce more avocados once trade is introduced. C) avocado prices in the United States will fall once trade begins. D) soybean prices in Mexico will rise once trade begins.

C) avocado prices in the United States will fall once trade begins.

A major reason for creating the European Monetary System was to A) create a single currency. B) unify banking laws and permit cross-border investment. C) avoid competitive devaluations. D) reduce the costs of changing currencies. E) eliminate the need for central banks.

C) avoid competitive devaluations.

Suppose the exchange rates between the United States and Canada are in long-run equilibrium as defined by the idea of purchasing power parity. If the law of one price holds perfectly, then differences between U.S. and Canadian rates of inflation would A) have no effect on nominal exchange rates. B) be completely offset by changes in the real exchange rate. C) be completely offset by changes in the nominal exchange rate. D) violate the conditions for the law of one price. E) lead to a change in the real purchasing power of each country's currency when it is converted to the other country's currency.

C) be completely offset by changes in the nominal exchange rate.

When expansionary fiscal and monetary policies are joined with a ________ exchange rate system, the various components of economic policy often interact in ways that lead to a crisis followed by a steep recession. A) fixed B) floating C) crawling peg D) flexible

C) crawling peg

High tariffs on intermediate inputs A) increase the effective rate of protection on final goods. B) have no impact on the effective rate of protection on final goods. C) decrease the effective rate of protection on final goods. D) lower the nominal rate of protection on final goods. E) raise the nominal rate of protection on final goods.

C) decrease the effective rate of protection on final goods.

A fixed exchange rate system crisis may be accompanied or followed by A) unexpected gains of international reserves. B) revaluation of a currency. C) devaluation of a currency. D) gains in comparative advantage. E) deflationary pressures within the country.

C) devaluation of a currency.

As Europe explored monetary union, evidence to date suggests that increased variability in exchange rates A) reduces foreign trade and investment. B) increases foreign trade and investment. C) does not seem to have an impact on foreign trade and investment. D) hurts foreign investment but not trade. E) hurts foreign trade but not investment.

C) does not seem to have an impact on foreign trade and investment.

A firm that buys foreign exchange in order to take advantage of higher foreign interest rates is A) speculating. B) demonstrating purchasing power parity. C) engaging in interest rate arbitrage. D) responding to fluctuations in the business cycle. E) ignoring the nominal rate of exchange.

C) engaging in interest rate arbitrage.

Intermediate inputs are A) goods used for household consumption only. B) goods used for government consumption only. C) goods purchased by one business from another to use in production. D) goods purchased by foreigners. E) raw materials used in the production process.

C) goods purchased by one business from another to use in production.

Using the HO model, assume that the United States is capital abundant and Mexico is labor abundant. If soybeans are capital intensive and avocados are labor intensive, it would be reasonable to expect the United States to A) specialize completely in soybean production. B) specialize completely in avocado production. C) increase soybean production, but still produce some avocados. D) increase avocado production, but still produce some soybeans.

C) increase soybean production, but still produce some avocados.

Suppose that Brazil is capital abundant and Chile is natural resource abundant. If timber is natural resource intensive and computers are capital intensive, then according to the Heckscher-Ohlin Theorem, Chile should export goods that A) intensively use labor input. B) intensively use capital input. C) intensively use natural resources. D) use capital and labor in about equal proportions.

C) intensively use natural resources.

Interindustry trade refers to A) international trade of products made within the same industry. B) domestic trade of products made within the same industry. C) international trade of products made across different industries. D) the exchange of similar items that are differentiated. E) None of the above.

C) international trade of products made across different industries.

Average tariff rates are highest for A) high-income countries. B) middle-income countries. C) low-income countries. D) industrialized countries.

C) low-income countries.

The economic philosophy that favors strict limits on imports and strong support for exports is called A) zero sum. B) autarky. C) mercantilism. D) comparative advantage. E) absolute advantage.

C) mercantilism.

The single currency project in the EU will be most successful if European labor is relatively A) immobile. B) immobile and business cycles are not synchronized. C) mobile and business cycles are synchronized. D) mobile and business cycles are not synchronized. E) mobil

C) mobile and business cycles are synchronized.

Suppose that the United States decided to subsidize a major competitive effort by the semiconductor industry to bring to market the next generation computer chip. Under the current rules of the WTO and GATT, the U.S. effort is A) allowed as long as the subsidies are a small percent of the total cost. B) allowed as long as the subsidies do not involve a direct payment to the industry. C) not allowed. D) neither allowed nor disallowed. E) allowed as long as the subsidies do not succeed in creating a new product.

C) not allowed.

An internal economies of scale is defined as A) one whose size or scale effects are not located in the firm, but in the industry. B) one with falling costs over a specific level of output. C) one with falling costs over a relatively large range of output. D) one with falling costs over a relatively large range of output, but definite declining profits.

C) one with falling costs over a relatively large range of output.

Based on Table 3.1, trade between the United States and Mexico will occur as long as the relative price of shoes is between A) three computers and one computer. B) three computers and two computers. C) one-half computer and one-third computer. D) six computers and three computers. E) None of the above.

C) one-half computer and one-third computer.

After implementation of the Single European Act, value added taxes in the EU were A) completely harmonized. B) unchanged. C) partially harmonized with minimum and maximum permissible values set by the EU. D) eliminated except on a few items in each country. E) cut in half in order to increase the role of corporate taxes.

C) partially harmonized with minimum and maximum permissible values set by the EU.

Purchasing power parity (PPP) measurements of income are a way to make international comparisons by correcting for national differences in A) unemployment. B) inflation. C) prices of goods and services. D) economic growth. E) government subsidies.

C) prices of goods and services.

Empirical tests of the theory of comparative advantage have provided A) strong support for both the Ricardian and Heckscher-Ohlin models. B) mixed support for the Ricardian model and strong support for the Heckscher-Ohlin model. C) strong support for the Ricardian model and mixed support for the Heckscher-Ohlin model. D) mixed support for both Ricardian and Heckscher-Ohlin models. E) no support for either the Ricardian or the Heckscher-Ohlin models.

C) strong support for the Ricardian model and mixed support for the Heckscher-Ohlin model.

Wage inequality has been on the rise in virtually all high-income industrial economies since the 1970s. The causes are probably numerous, but the leading explanation for the greatest share of the increase in inequality is A) the growth of trade with developing countries. B) the growth of trade with other high income industrial countries. C) technological change which increased the relative demand for skilled workers. D) technological change which increased the relative demand for unskilled workers. E) technological change which decreased the relative demand for skilled workers.

C) technological change which increased the relative demand for skilled workers.

Most economists attribute the growing income inequality in the United States to A) trade. B) macroeconomic policies. C) technological change. D) changing values. E) taxes.

C) technological change.

Fiscal policy is A) the selling of government bonds by the Treasury. B) the deliberate manipulation of the money supply designed to affect the interest rate. C) the deliberate manipulation of taxation and spending designed to affect the economy. D) the selling of foreign exchange reserves designed to change the exchange rate. E) All of the above.

C) the deliberate manipulation of taxation and spending designed to affect the economy.

The production side efficiency loss of a tariff is caused by A) confusion about prices when a tariff is imposed. B) higher profits gained by foreign producers. C) the expansion of relative inefficient domestic production. D) the contraction of domestic consumption. E) the increase in government revenue.

C) the expansion of relative inefficient domestic production.

The Stolper-Samuelson Theorem predicts A) the level of productivity in export industries. B) which factors are abundant. C) the income distribution effects of trade. D) which goods will be exported. E) the importance of intraindustry trade.

C) the income distribution effects of trade.

All else equal and given the current system of exchange rates, if the United States enters a period of exceptionally strong growth, A) the pressure on the dollar is to revalue. B) the pressure on the dollar is to devalue. C) the pressure on the dollar is to depreciate. D) the pressure on the dollar is to appreciate. E) Both A and D.

C) the pressure on the dollar is to depreciate.

The biggest disadvantage of a fixed exchange rate is the A) increased probability of high inflation. B) tradeoff between supporting the exchange rate and adjusting the trade balance. C) tradeoff between supporting the exchange rate and maintaining economic growth. D) increased probability of a trade deficit. E) tradeoff between supporting the exchange rate and maintaining a balanced budget.

C) tradeoff between supporting the exchange rate and maintaining economic growth.

The most important participants in foreign exchange markets are ________.

Commercial Banks

How does the growth in the daily volume of foreign currency transactions compare with the growth rate of the global economy?

Currency transactions are growing more rapidly.

Suppose Mexico can produce 5 autos or 10 corn. Suppose the United States can produce 4 autos or 20 corn. If opportunity costs are constant for both countries, which of the following would NOT be a potential terms of trade? A) 1 auto for 3 corn B) 1 auto for 4 corn C) 1 corn for 1/3 of an auto D) 1 corn for 1 auto

D) 1 corn for 1 auto

Based on Scenario 6.1 above, if a tariff of 20 percent is placed on imports of dining room tables, and another tariff of 50 percent is placed on imports of wood and parts, then the effective rate of protection on tables made in the United States is A) 70 percent. B) 50 percent. C) 20 percent. D) 12.5 percent. E) 0 percent.

D) 12.5 percent.

Which of the following statements is FALSE? A) Comparative advantage is the principle upon which trade patterns are based. B) Opportunity cost measures the real cost to a country of producing a certain product. C) The gains from trade are the result of differences in opportunity cost and comparative advantage. D) A country that possesses an absolute advantage will always have a comparative advantage. E) Comparative advantage is necessary and sufficient for trade.

D) A country that possesses an absolute advantage will always have a comparative advantage.

Which of the following is true? A) If an exchange rate is allowed to vary across a fixed basket of currencies, it is called a hard peg. B) If an exchange rate is not allowed to vary against the target currency, it is called a soft peg. C) If an exchange is only allowed to fluctuate within a set band, it is considered to be a flexible exchange rate system. D) A soft peg is when a currency's exchange rate is only allowed to fluctuate within a set band. E) Any exchange rate policy other than completely flexible exchange rate systems is extremely uncommon today for currencies.

D) A soft peg is when a currency's exchange rate is only allowed to fluctuate within a set band.

Large countries can improve their welfare by levying a tariff only if it does not A) encourage rent seeking elsewhere in the economy. B) discourage innovation. C) lead to retaliation by the nation's trading partners. D) All of the above. E) None of the above.

D) All of the above.

The effects of tax cuts or government spending dissipate and each additional change in consumption and income becomes smaller and smaller because A) some of the increase in income will be lost through taxation. B) some of the increase is saved and does not result in an increase in consumer demand. C) some of the increase in consumption will be an additional demand for imported goods. D) All of the above. E) None of the above.

D) All of the above.

All else equal, if Canada raises its interest rates, A) the dollar depreciates. B) the U.S. demand for Canadian dollars increases. C) the Canadian supply of Canadian dollars increases. D) Both A and B. E) Both A and C.

D) Both A and B.

Suppose that Canada can produce 15 timber or 3 film and Mexico can produce 9 timber or 3 film. Suppose that opportunity costs are constant. Which of the following is FALSE? A) Canada has an absolute advantage in timber production. B) Mexico has a comparative advantage in film production. C) The opportunity costs for producing timber are lower in Canada than in Mexico. D) Canada and Mexico would find trade mutually advantageous at a ratio of one unit of film to six units of timber.

D) Canada and Mexico would find trade mutually advantageous at a ratio of one unit of film to six units of timber.

If the case study on U.S. / China trade is correct in its analysis of factor abundance, A) Chinese capital owners should see their income rise as trade increases. B) U.S. skilled labor inputs should see their incomes fall as trade increases. C) U.S. capital owners should see their income fall as trade increases. D) Chinese unskilled labor should see their income rise as trade increases.

D) Chinese unskilled labor should see their income rise as trade increases.

Which of the following is FALSE about issues/negotiations in the Doha Development agenda? A) It is intended to deal with economic development issues and trade barriers facing developing countries that were not adequately addressed in the Uruguay Round. B) Many developing countries are upset with the levels of tariffs and other barriers that industrialized countries use to protect agriculture, clothing and textiles. C) Industrialized countries want developing countries to reduce their tariffs, which on average are higher than the rates of richer countries. D) Developing countries don't use tariffs, and they want higher income countries to follow their model.

D) Developing countries don't use tariffs, and they want higher income countries to follow their model.

Which of the following is FALSE? A) At the end of the twentieth century, more and more traded goods and services incorporated specialized knowledge and unique ideas. B) Pharmaceuticals, computer hardware, telecommunications equipment, and other high technology products are valuable because of the innovation and research they incorporate. C) Software, movies, music, and other artistic expressions are valued for their creativity. D) Developing countries usually strongly advocate the protection of intellectual property rights. E) The protection given to creators and innovators varied greatly internationally until standardization began with the signing of the TRIPs agreement.

D) Developing countries usually strongly advocate the protection of intellectual property rights.

Intraindustry trade is characterized by what two features of the industry and market? A) Diseconomies of scale and homogeneous products B) Non-tariff barriers and large-scale foreign investment C) Quota auctions and low effective rates of protection D) Economies of scale and differentiated products E) Government subsidies and industrial policy

D) Economies of scale and differentiated products

The single most important factor in the 1992 crisis of the EMS was A) ratification of the Single European Act. B) the near failure of the Maastricht Treaty. C) the fall in unemployment rates throughout the EEC. D) German reunification efforts. E) the outbreak of war in the former Yugoslavia.

D) German reunification efforts.

Which of the following is NOT a likely result of intraindustry trade based on internal economies of scale? A) Job creation at domestic firms entering international trade B) Lower prices for the domestic consumers of the product now being traded C) Increased sales and lower per unit costs for the firm doing the exporting D) Higher prices for the exported product

D) Higher prices for the exported product

Which of the following may NOT help avoid a financial crisis? A) Maintaining credible and sustainable fiscal policies B) Regulation and supervision of the financial system C) Disclosure of timely information to lenders, investors, and depositors about key economic variables such as the central bank's holding of international reserves D) Immediately bailing out financial intermediaries and standing ready to bail out others in case a financial crisis occurs E) Maintaining credible and sustainable monetary policies

D) Immediately bailing out financial intermediaries and standing ready to bail out others in case a financial crisis occurs

Which of the following is FALSE? A) Economies of scale may be internal, external, or both. B) With internal economies of scale, the gains from trade include a wider selection of consumer choices and lower prices. C) With external economies of scale, the gains from trade are less certain since, in theory, they can lock in production in a less efficient country and prevent the development of production in a more efficient country. D) Internal economies of scale lead firms to regionally concentrate their industry.

D) Internal economies of scale lead firms to regionally concentrate their industry.

Based on Table 3.1, which country or countries has an absolute advantage and a comparative advantage in shoes? A) Mexico has an absolute and comparative advantage in shoes. B) The United States has an absolute and comparative advantage in shoes. C) The United States has a comparative advantage, and Mexico has an absolute advantage in shoes. D) Mexico has a comparative advantage, and the United States has an absolute advantage in shoes. E) There is not enough information to tell.

D) Mexico has a comparative advantage, and the United States has an absolute advantage in shoes.

Which of the following would NOT be a cause for an increased American demand for the Mexican peso? A) The United States having lower interest rates than Mexico B) Increased American demand for Mexican goods C) The expectation by speculators that the value of the peso is edging up D) More economic expansion in the United States E) None of the above.

D) More economic expansion in the United States

Which nation is NOT one of the current members of the European Union? A) Greece B) Germany C) Sweden D) Norway

D) Norway

According to purchasing power parity, which of the following is FALSE about an overvalued dollar compared to the Japanese yen? A) U.S. merchants would be motivated to import more Japanese goods. B) Japanese merchants would tend to export more to the United States. C) Prices in the United States would tend to fall. D) Over the long term, the exchange rate would fall. E) Prices in Japan would tend to rise.

D) Over the long term, the exchange rate would fall.

Which of the following is NOT correct about the effects of a tariff on an imported product? A) Tariffs benefit domestic producers by raising price and domestic output. B) Tariffs increase government revenue. C) Tariffs mean higher prices and less consumption for consumers of the product. D) Tariffs increase the efficiency of how resources are allocated.

D) Tariffs increase the efficiency of how resources are allocated.

Which of the following is TRUE? A) Adam Smith proposed the theory of comparative advantage as the basis for trade in The Wealth of Nations. B) David Ricardo proposed the theory of absolute advantage as the basis for trade. C) Absolute advantage is based on comparing the opportunity costs of trading partners. D) The Ricardian model assumes labor is perfectly mobile.

D) The Ricardian model assumes labor is perfectly mobile.

Which of the following would be a deadweight loss from a tariff? A) The shift of consumer surplus to government B) The increase in producer surplus C) The decrease in consumer surplus D) The decrease in consumer surplus due to a drop in consumption E) All of the above.

D) The decrease in consumer surplus due to a drop in consumption

Which of the following would NOT be associated with the LATE PHASE of the product cycle? A) Consumption in high income countries begins to exceed production. B) Increasing share of output is moving to developing countries where abundant low skilled and semi-skilled labor keep production costs low. C) Consumption continues to grow in low income countries. D) There is experimentation and improvement in design and manufacturing.

D) There is experimentation and improvement in design and manufacturing.

In which way are tariffs different from quotas? A) They reduce the volume of imported products. B) They raise the price of the imported products to consumers. C) They increase the domestic quantity supplied of the product. D) They raise government revenue.

D) They raise government revenue.

Which of the following would NOT be a reason why developed nations would try to coordinate their macroeconomic policies? A) To achieve a desirable level of world economic growth B) To avoid imposing a disproportionate burden on one major country in its attempt to help other world economies C) To stimulate production in other countries using the higher incomes generated by policy coordination D) To coordinate retaliatory policies on developing countries' trade barriers E) None of the above. That is, A, B, C, and D are all correct.

D) To coordinate retaliatory policies on developing countries' trade barriers

The European Union became an economic union with the implementation of which treaty? A) Treaty of Rome B) Single Europe Treaty C) Delor's Agreement D) Treaty on European Union E) Schengen Agreement

D) Treaty on European Union

Which of the following is NOT one of the factors your text identifies that could affect the size of the multiplier? A) What sector of the economy receives the initial increase in spending B) How the spending is financed C) Whether the economy is in recession or at full employment D) Whether the economy is in autarky equilibrium

D) Whether the economy is in autarky equilibrium

Which of the following is NOT one of the determinants of the gains of adopting a single currency? A) A well-synchronized business cycle involving all member countries B) The possibility of factors of production to freely move across borders C) The willingness and ability of member countries to design policies to address regional imbalances that may develop D) Widening the common market by allowing other countries to join E) None of the above.

D) Widening the common market by allowing other countries to join

All of the following took place during the Great Depression except A) increase in unemployment from about 3.4 percent to about 25 percent and a decrease in real GDP by about 30 percent between 1929 in 1933. B) an increase in taxes because of the fear that budget deficits would undermine business confidence. C) a fall in the money supply by more than 30 percent. D) a rise in inflation during the early 1930s. E) the stock market crashed by about one-third in October of 1929.

D) a rise in inflation during the early 1930s.

Suppose the Asian financial crisis decreased U.S. exports. In the aggregate demand/aggregate supply model, this would be represented as A) a shift to the right of aggregate supply, which would result in more production for the U.S. economy. B) a shift to the left of aggregate supply, which would result in less production for the U.S. economy. C) a shift to the right of aggregate demand, leading to more spending and production in the U.S. economy. D) a shift to the left of aggregate demand, leading to less spending and production in the U.S. economy.

D) a shift to the left of aggregate demand, leading to less spending and production in the U.S. economy.

All of the following are possible outcomes of a financial crisis except A) bank failings and disintermediation. B) decreases in investment. C) a recession. D) a sure competitive advantage accompanied by a comparative advantage. E) depreciation or devaluation of a currency.

D) a sure competitive advantage accompanied by a comparative advantage.

An example of expansionary fiscal policy would be A) a decrease in government spending to reduce budget deficits. B) an increase in tax collection to reduce budget deficits. C) a decrease in interest rates to help stimulate the economy. D) an increase in government spending on infrastructure to create jobs and improve the economy. E) an increase in interest rates to encourage private savings.

D) an increase in government spending on infrastructure to create jobs and improve the economy.

Government spending and taxes A) do not change aggregate demand. B) are an important component of aggregate supply. C) do not play a big role in determining GDP. D) are a major determinant of aggregate demand. E) cannot affect the price level.

D) are a major determinant of aggregate demand.

Nominal rates of protection A) are always greater than effective rates of protection. B) are always smaller than effective rates of protection. C) refer to the tariffs placed on intermediate goods used to make the final good or service. D) cannot be negative.

D) cannot be negative.

The basis for free trade is the concept of A) absolute advantage. B) differences in natural resources and climate. C) differences in nominal wages. D) comparative advantage.

D) comparative advantage.

In the case of a small country, consumer surplus A) decreases less with a tariff than with an equivalent quota. B) decreases less with a quota than with an equivalent tariff. C) is not changed by tariffs or quotas. D) decreases the same with tariffs and equivalent quotas. E) increases more with quotas.

D) decreases the same with tariffs and equivalent quotas.

The Basel Capital Accord does NOT include A) requiring bank owners to invest into and have some capital ownership in the banks they own. B) supervision of banks by an oversight board. C) information disclosure designed to encourage market discipline. D) denying access to foreign capital by a country that defaults on its international loans. E) None of the above.

D) denying access to foreign capital by a country that defaults on its international loans.

The principle of subsidiarity is a way to A) divide power between local governments and unions. B) provide support for industries in decline. C) provide support for industries under pressure from foreign competition. D) divide power between national governments and the EU. E) divide EU tax money among the member countries.

D) divide power between national governments and the EU.

If a country has lower overall productivity levels than its trading partners, then it will A) be unable to export. B) have a trade deficit. C) not be able to obtain gains from trade. D) have a lower standard of living than its trading partners. E) All of the above.

D) have a lower standard of living than its trading partners.

Economic restructuring that takes place as a result of opening to trade with other countries A) contradicts the idea of gains from trade. B) causes some trading activity to be zero sum. C) worsens the nation's allocation of resources. D) improves the nation's allocation of resources. E) is a highly unlikely event.

D) improves the nation's allocation of resources.

In the case of a small country, producer surplus A) increases more with a tariff than with an equivalent quota. B) increases more with a quota than with an equivalent tariff. C) is not changed by tariffs or quotas. D) increases the same with tariffs and equivalent quotas. E) increases more with quotas.

D) increases the same with tariffs and equivalent quotas.

After trade opens, the short run impact on the income of the variable factor will be A) a decrease. B) an increase. C) zero. D) indeterminate, depending on the consumption pattern of the owners of the variable factor. E) indeterminate, depending on the productivity of the variable factor.

D) indeterminate, depending on the consumption pattern of the owners of the variable factor.

One drawback to a single currency is that A) the exchange rate is more volatile. B) bond markets are larger and therefore harder to control. C) exporters and importers have fewer choices about how they will receive and make payments. D) individual nations cannot use monetary policy to stabilize the economy. E) foreign currency is more expensive.

D) individual nations cannot use monetary policy to stabilize the economy.

U.S. opponents of NAFTA argued that the agreement will hurt the United States. In their view, NAFTA's greatest harm will be its A) harmonization of labor policies. B) harmonization of environmental policies. C) harmonization of the use of pesticides and herbicides. D) job destruction and downward pressure on U.S. wages. E) harmonization of product safety standards.

D) job destruction and downward pressure on U.S. wages.

The Bretton Woods exchange rate system was an example of a A) target zone. B) managed float. C) pure gold standard. D) modified gold standard. E) floating exchange rate system.

D) modified gold standard.

Based on Table 3.1, the opportunity cost of a pair of shoes in the United States is A) three computers. B) two computers. C) one computer. D) one-half computer. E) None of the above.

D) one-half computer.

The United States' comparative advantage over Japan in the production of rock-n-roll music implies that (for a similar quality of music) the A) opportunity cost of production is less in Japan. B) absolute cost of production is less in the United States. C) absolute cost of production is less in Japan. D) opportunity cost of production is less in the United States. E) None of the above.

D) opportunity cost of production is less in the United States.

The new GATS and TRIPS are separate agreements negotiated within the WTO framework as part of the Uruguay Round that apply to A) services and aircraft. B) services and transportation. C) agriculture and textiles. D) services and intellectual property. E) textiles and transportation.

D) services and intellectual property.

Internal economies of scale means that A) firms are experiencing lower average production costs due to a geographical concentration of firms in their industry that make it cheaper and easier to hire highly specialized workers and inputs. B) firms will have lower profits after international trade begins, because costs will be higher than when they just focused on the domestic market. C) consumers will have less choices once trade begins, because firms will be squeezed out of the market. D) simply expanding the size of the market the firm serves reduces overall per unit costs, since the firm can spread costs over more output.

D) simply expanding the size of the market the firm serves reduces overall per unit costs, since the firm can spread costs over more output.

The main area of economic activity that Canada chose not to open to free trade is A) agriculture. B) oil and gas. C) lumber. D) the cultural industries. E) financial services.

D) the cultural industries.

Efficiency losses are A) deadweight losses caused by consumers being prevented by tariffs from buying products at the world price, products that they value more highly than that price. B) the total loss in consumer surplus from a tariff. C) the increase in producer surplus that is created by a tariff. D) the deadweight loss that is created because domestic firms have to charge higher prices to produce units of output than foreign firms would have to charge.

D) the deadweight loss that is created because domestic firms have to charge higher prices to produce units of output than foreign firms would have to charge.

All of the following are symptoms of definite and indefinite macroeconomic imbalances except A) large budget deficits. B) an overvalued currency. C) a current account deficit. D) the discovery of emerging markets by financial investors who want to diversify their portfolios. E) inflationary pressures.

D) the discovery of emerging markets by financial investors who want to diversify their portfolios.

In order to protect against foreign exchange risk, firms can use A) the spot market for foreign exchange. B) interest rate arbitrage. C) purchasing power parity. D) the forward market for foreign exchange. E) the J-curve.

D) the forward market for foreign exchange.

Using the specific factors model, assume that strawberry production requires the specific factor of land, tractor production requires the specific factor of capital, and labor is variable. If the United States is capital abundant compared to Mexico, and Mexico is land abundant compared to the United States, then in the short run with trade we would expect A) the income of U.S. land owners to increase. B) the income of U.S. workers to increase. C) the income of Mexican workers to increase. D) the income of Mexican land owners to increase.

D) the income of Mexican land owners to increase.

All of the following are true EXCEPT A) trade between two nations reduces their opportunity costs. B) trade makes nations dependent on each other. C) trade between nations will not benefit all citizens. D) the principle of comparative advantage does not apply to countries with extremely limited resources. E) specialization according to comparative advantage can make both countries better off.

D) the principle of comparative advantage does not apply to countries with extremely limited resources.

All of the following involve a moral hazard problem except A) an individual driving carelessly after buying a comprehensive insurance policy for a Ford Pinto. B) the IMF bailing Mexico out of a financial crisis, with promises to do the same for other nations that might face financial problems. C) making regular visits to your doctor because you know that you have full healthcare coverage. D) the requirement of banking institutions that owners invest a substantial portion of their own capital in their bank. E) membership in FDIC (Federal Deposit Insurance Corporation) by your local bank.

D) the requirement of banking institutions that owners invest a substantial portion of their own capital in their bank.

If the price of a good rises, then the effect on the income of the factors that are used intensively in its production will be A) to raise income by an absolute amount that is less than the rise in prices. B) to raise income by an absolute amount that is more than the rise in prices. C) to raise income by a smaller percentage than the rise in prices. D) to raise income by a greater percentage than the rise in prices. E) to cause income to fall.

D) to raise income by a greater percentage than the rise in prices.

The Heckscher-Ohlin Theorem predicts A) who benefits and who loses from trade. B) which factors are abundant. C) the income distribution effects of trade. D) which goods will be exported. E) the importance of intraindustry trade.

D) which goods will be exported.

One area of labor issues that the labor side agreement to NAFTA does not open to foreign consultation or investigation is A) the use of child labor. B) worker exposure to unsafe conditions. C) minimum wages. D) worker's rights to organize. E) worker exposure to toxic hazards.

D) worker's rights to organize.

Which economist introduced the simple trade model and the concept of trade based on comparative advantage?

David Ricardo

What is the source of comparative advantage in the Heckscher-Ohlin model?

Differences in factor endowments

Which of the following is NOT a problem in the implementation of industrial policies? A) Choosing the industry to target B) Knowing the optimum amount of resources to provide the targeted industry C) The encouragement of rent seeking by firms in other industries D) The benefits are partly captured by foreign firms. E) All of the above are problems.

E) All of the above are problems.

An increase in the U.S. demand for the Mexican peso A) causes an increase in the U.S. dollar price of a Mexican peso. B) causes the Mexican peso to appreciate. C) causes the U.S. dollar to depreciate. D) causes Mexican goods to be relatively more expensive. E) All of the above.

E) All of the above.

Market failures occur whenever A) private returns may be greater than social returns. B) social returns may be greater than private returns. C) the free market produces less than what is socially optimal. D) monopolies exist in a market. E) All of the above.

E) All of the above.

One characteristic of a financial crisis caused by macroeconomic imbalances is that it A) may or may not be predictable. B) will occur eventually even though its timing is unpredictable. C) may be caused by expansionary fiscal policies accompanied by high budget deficits. D) may be caused by high deficits financed by increases in the money supply. E) All of the above.

E) All of the above.

The IMF conditionality may include A) changes in the fiscal and monetary policies of the country facing the financial crisis. B) changes in the exchange rate policies. C) regulating and restructuring the financial sector of the economy of the country in crisis. D) structural policies affecting international trade and public enterprises. E) All of the above.

E) All of the above.

An American firm that buys foreign exchange because its managers expect the dollar to depreciate is A) increasing the supply of foreign exchange. B) increasing the demand for foreign exchange. C) speculating. D) Both A and B. E) Both B and C.

E) Both B and C.

The Smithsonian Agreement of 1971 was hailed by President Nixon as a fundamental reorganization of the international monetary system. In fact, what it accomplished was A) the revaluation of the dollar. B) the devaluation of the dollar. C) the reduction of the gold content of the dollar. D) the elimination of gold backing for the dollar. E) Both B and C.

E) Both B and C.

The primary legislative branch of the European Union is called the A) European Commission. B) European Parliament. C) European Court of Justice. D) European Council of Ministers. E) Council of the European Union.

E) Council of the European Union.

Which of the following is NOT a proposition of the Heckscher-Ohlin model? A) A country has a comparative advantage in the production of that commodity which uses more intensively the country's more abundant resource. B) The effect of international trade is to tend to equalize factor prices between the trading nations. C) If Mexico is an unskilled labor abundant country, then Mexico has a comparative advantage in the production of goods that use unskilled labor more intensively. D) If the United States is a skilled labor abundant country, then the United States has a comparative advantage in the production of goods that use skilled labor more intensively. E) Countries will completely specialize in the product in which they have a comparative advantage if free trade is allowed to occur.

E) Countries will completely specialize in the product in which they have a comparative advantage if free trade is allowed to occur.

Starting from a balanced budget, which of the following would NOT cause a deficit? A) A decrease in taxes B) An increase in spending of goods and services C) An increase in transfer payments D) A 50 percent increase in spending accompanied by a 40 percent increase in taxes E) None of the above.

E) None of the above.

Which of the following is FALSE concerning the long run? A) Economists believe that fiscal and monetary policies have no permanent effects on the economy. B) Economists more or less agree that the economy tends to fluctuate around the level that is consistent with full employment. C) In the long run, the unemployment rate returns to its normal level. D) The current account must tend toward balance in the long run. E) None of the above.

E) None of the above.

Which of the following is NOT likely to occur when a bank fails? A) Everyone that deposits money in the bank loses all or a portion of their money, unless the country has a functioning deposit insurance system. B) The loss of savings (or the feared loss of savings) causes households to cut back on consumption, which spreads the recessionary effect wider through the country. C) Unaffected banks may stop making loans as they take a cautious approach, slowing or stopping new investment. D) Layoffs occur and the economy falls deeper into a downward spiraling inflation. E) Other banks make too many loans to make up for the loans not made by the failed bank, kicking off a cycle of stimulation and inflation.

E) Other banks make too many loans to make up for the loans not made by the failed bank, kicking off a cycle of stimulation and inflation.

Which of the following is NOT part of the European Union? A) Luxembourg B) Austria C) Portugal D) Greece E) Switzerland

E) Switzerland

Which of the following is FALSE? A) Tariffs are a relatively easy tax to administer and often form an important part of revenue for low-income countries. B) Taxes on income, sales, and property require more complex accounting systems than do tariffs. C) Low-income countries often have large informal markets with the sales of many goods and services not being recorded, which makes it difficult to apply many kinds of taxes. D) Taxes on income and property run into powerful interest groups in low-income countries, which work to prevent the creation or payment of these taxes. E) Tariffs are not an attractive tax option for most low-income countries, so they mostly rely on quota licenses for revenue.

E) Tariffs are not an attractive tax option for most low-income countries, so they mostly rely on quota licenses for revenue.

Which of the following does NOT indicate intraindustry trade? A) Trading Jeeps for Toyotas B) Trading Boeing airplanes for Airbus airplanes C) Trading Bush beer for Heineken beer D) Trading Japanese-made films for Hollywood-made films E) Trading oil for trucks

E) Trading oil for trucks

Suppose that the United States decided to subsidize a major research and development effort by U.S. firms in the semiconductor industry. Under the current rules of the WTO and GATT, the U.S. effort is A) allowed as long as the subsidies are a small percent of the total cost. B) allowed as long as the subsidies do not involve a direct payment to the industry. C) not allowed. D) neither allowed nor disallowed. E) allowed as long as the subsidies are for developing a "precompetitive" technology.

E) allowed as long as the subsidies are for developing a "precompetitive" technology.

When an individual or firm in the United States requests that a bank sell foreign exchange, the bank will probably A) call a foreign bank and arrange a purchase. B) call the central bank and arrange a purchase. C) call another bank customer with foreign exchange holdings. D) call another domestic bank and arrange a purchase. E) call a foreign exchange broker and arrange a purchase.

E) call a foreign exchange broker and arrange a purchase.

In economic terms, tariffs are preferred to quotas because A) domestic manufacturers gain more producer surplus. B) there is less loss of consumer surplus. C) tariffs are easier to administer. D) quotas create a greater production inefficiency. E) given the way quotas are usually administered, tariffs cause a smaller net national welfare loss.

E) given the way quotas are usually administered, tariffs cause a smaller net national welfare loss.

All of the following issues were discussed as options for reforming the international financial architecture except A) how high an interest rate the lender of last resort should charge when it makes loans. B) the length of the payback period. C) the size of the loans. D) the moral hazard problem associated with a lender of last resort. E) if the lender of last resort (i.e., the IMF) should consult and collaborate with other international institutions such as the United Nations and the WTO.

E) if the lender of last resort (i.e., the IMF) should consult and collaborate with other international institutions such as the United Nations and the WTO.

One reason that a large share of the trade between high-income industrial economies is intraindustry trade is because A) it is more advantageous than interindustry trade. B) high-income industrial economies produce in the first stage of the product cycle. C) high-income industrial economies are wealthier than developing countries. D) high-income industrial economies have dissimilar resource endowments in absolute terms. E) it allows firms to take advantage of internal economies of scale.

E) it allows firms to take advantage of internal economies of scale.

Competition between the United States and Mexico is A) equivalent to the competition between two giant corporations. B) a struggle over which country will get the best jobs. C) unfair if wages in Mexico are lower than in the United States. D) a struggle over which country will keep the most advanced technology. E) not a meaningful way to analyze trade.

E) not a meaningful way to analyze trade.

Under NAFTA, environmental standards are A) harmonized around Mexican rules. B) harmonized around U.S. rules. C) harmonized around Canadian rules. D) harmonized around a combination of the rules in all three countries. E) not harmonized.

E) not harmonized.

Along the aggregate supply curve A) the horizontal part represents a situation where the economy is operating above full employment levels. B) inflation would be a primary concern along the horizontal part of the aggregate supply curve. C) idle resources, such as labor and capital, would be a feature of the vertical section of the aggregate supply curve. D) the horizontal section of the aggregate supply curve represents the limit of production. E) the middle, upward-sloping part of the aggregate supply curve would be associated with a growing economy that experienced increased prices from resources that are becoming relatively scarce.

E) the middle, upward-sloping part of the aggregate supply curve would be associated with a growing economy that experienced increased prices from resources that are becoming relatively scarce.

Consumer surplus is equal to the area A) under the demand curve and above the supply curve. B) above the supply curve and below the price line. C) under the demand curve. D) under the supply curve. E) under the demand curve and above the price line.

E) under the demand curve and above the price line.

Intraindustry trade is characterized by what two features?

Economies of scale and differentiated products

The spot rate is the rate at which foreign currencies will be exchanged a specified number of days in the future

FALSE

A country is likely to be better off in the long run if it pursues self sufficiency.

False

A country that experiences higher real interest rates than other countries would expect its currency to depreciate.

False

A nation must have an absolute advantage in order to have a comparative advantage in producing a good or service.

False

Adam Smith created the theory of comparative advantage.

False

All individuals and firms in a country must gain from trade in order for it to be beneficial to the nation.

False

Deadweight losses are the only potential cost associated with tariffs, which is why they are preferred to quotas.

False

Economies of scale are an important determinant of comparative advantage based trade.

False

If Juana contracts to buy U.S. office equipment in U.S. dollars and her domestic currency depreciates against the U.S. dollar between the time the contract is signed and the bill is paid, she will wind up paying less for the equipment because she stayed in the spot market.

False

If countries have similar factor endowments and productivities, their trade is likely to be interindustry.

False

If more European and Japanese firms want to build factories and expand their offshore investments in the United States, the supply of U.S. dollars on foreign exchange markets will increase as a result of this investment activity.

False

If the Costa Rican colone is expected to depreciate in the future, it will temporarily appreciate as people move to take advantage based on this expectation.

False

If the effective rate of protection is greater than the nominal rate of protection, there must be tariffs on intermediate products.

False

Interindustry trade is not based on comparative advantage since it consists of the export and import of similar countries and mostly between countries that have similar productivity, technology, and factor endowments.

False

Internationally, the TRIPS agreement is uniformly regarded as a positive step for world prosperity.

False

Intraindustry trade tends to be more controversial than interindustry trade.

False

Nations get significant advantages if they have a single, valuable, natural resource, with little downside risk.

False

OLI theory is a direct contradiction of trade theory, especially trade theory based on comparative advantage.

False

Offshoring by domestic firms causes job losses not job expansion in the home market.

False

Since the mid-1980s tariff rates in most nations have risen

False

Speculation would involve using forward contracts and options to reduce the exchange rate risk on future foreign exchange transactions.

False

Tariff revenue is an important source of operating revenue for many governments of high income countries

False

The L in OLI theory stands for loyalty, and this factor makes it more difficult for firms to substitute foreign operations for domestic as they fear a loss of sales due to negative publicity.

False

The United States does not use subsidies as part of its policies.

False

The United States has the largest percentage of foreigners in its overall population of any nation.

False

The bulk of offshoring is vertical, relating to producing a component piece in an overall supply chain production.

False

The gains from trade rely on overall productivity (absolute advantage).

False

The opportunity cost of producing in low-income, developing countries rises over the product cycle, according to theory.

False

The primary interest of firms engaging in offshoring is to find lower wages and to decrease production costs.

False

When Jeneva went to Costa Rica in July 2008, a U.S. dollar was worth 550 colones. If today a U.S. dollar is worth 650 colones, it means that the U.S. dollar has depreciated against the colone.

False

What are the three major types of quotas?

Formal quotas outright limit the quantity of imports; import licenses; voluntary export restraints

Which types of workers are most likely to favor lowering trade barriers in the United States?

Highly educated and more skilled

What did Adam Smith perceive was primarily responsible for improving standard of living?

Increased specialization in production

If the United States and Canada trade hydro-powered electricity for Hollywood films, what type of trade does this represent?

Interindustry

If General Motors imports parts from its plants in Canada and Mexico for finished trucks that it will sell across the NAFTA region, what type of trade does this represent?

Intrafirm

If countries have similar factor endowments and productivities, what type of trade are they most likely to have?

Intraindustry

If the United States and Mexico trade Budweiser for Modelo beer, what type of trade does this represent?

Intraindustry

Given the Republic of Korea's experience, what can we conclude about comparative advantage?

It can change over time. More skill and more capital can be developed as income rises, changing productivity.

What results has the Republic of Korea experienced from its change in policies?

It has successfully industrialized. Its index of openness is many times greater and incomes have grown almost 6 percent per year over the period.

How does rapid economic growth at home affect foreign exchange markets?

It increases home country demand for foreign currencies as it increases imports. The home currency depreciates.

In the simple trade model, what is assumed about labor?

It is perfectly mobile between the two industries within a nation.

When did NAFTA go into effect?

January 1, 1994

What is the largest center for currency trading?

London

Describe some of the key controversies regarding global cotton trade between high cost and low cost cotton producers.

Low cost cotton producers produce less cotton in total, but rely more on cotton exports for income. Their low incomes put them on the edge of survival and the fact that they make less in total to export makes cotton export revenues critical. High cost cotton producers depend much less on their cotton exports and have much higher incomes. High cost cotton producers are assisted through direct and indirect payments from their governments, tariffs on cotton imports, farm support programs including subsidized loans, insurance, marketing and promotion assistance, and revenue guarantees. These programs keep cotton production where it is less efficient and have the potential to harm living standards in developing nations and keep them from fully exploiting their comparative advantage in cotton.

What type of policies did Adam Smith attack in his book, An Inquiry into the Nature and Causes of the Wealth of Nations?

Mercantilism

The relocation of service industry functions to another country is called what?

Offshoring

How is offshoring of services different from past trade patterns?

Services were consumed where they were produced and not traded.

Forces inside a nation that cause people to think about leaving that nation are called what in migration theory?

Supply-push factors

Why is the U.S. putting less emphasis on multilateral and more emphasis on bilateral trade agreements?

Tariffs low, quotas mostly eliminated; Cold War is over; remaining trade issues are tough;, more participants makes multilateral negotiations difficult; bilateral can be testing ground for new arrangements

What three sources of revenue finance the EU budget?

Tariffs on goods entering EU; EU share of national value added taxes; payments from member countries based on the size of their economies

In the Heckscher-Ohlin model, what assumption is made about opportunity costs?

That opportunity costs increase

What does research thus far suggest about job loss and offshoring?

That outsourcing can be complementary and not just a substitute for domestic labor

When did the Republic of Korea change its economic policies?

The 1960s

Members of which European Union institution are popularly elected?

The European Parliament

What is productivity?

The amount of output from a unit of an input

Why are trade agreements between the U.S. and Mexico more controversial than trade agreements between the U.S. and Canada?

The difference in levels of economic development is greater which leads to larger political and social tensions.

If a nation is more productive than a trading partner, can it still gain from trade with that partner? Use the concepts of absolute and comparative advantage to explain.

The gains from trade do not rely on overall productivity (absolute advantage) but on differences in relative prices (comparative advantage). In producing a good or service, as long as a trading partner gives up fewer units of an alternate product, we can gain from trade with them.

If the forward rate is greater than the spot rate, what are markets signaling about their expectations for the future spot rates for the home currency?

The home currency is expected to depreciate over the maturity period of the forward contact.

Free trade may be good for a nation, but not for everyone in the nation. Explain why free trade is controversial and the list the justifications that proponents of trade adjustment assistance offer in support of those policies.

The nation as a whole is better off as long as the gains from the winners exceed the losses from the losers. Restructuring means that some industries are dying and workers and producers have to find other opportunities. This may be difficult for not only those directly involved in the industry, but also for the surrounding communities. Adjustment assistance states that since the nation as a whole benefits from trade, there are newly added resources that can be used for compensation. Second, there is a potential ethical obligation to assist those hurt by economic change. Third, compensation reduces the incentives for those hurt by trade to fight against free trade policies.

How is dollarization different from monetary union?

The nations do not share a central bank and monetary policy.

What are bilateral investment treaties?

They set out the rules that govern cross border investment and typically emphasize national treatment for foreign investors.

A country that creates competitive advantage where there are not comparative advantages misallocates its resources and has lower national well being.

True

A forward exchange market contract obligates the owner to make a trade at a specified exchange rate a fixed number of days in the future.

True

A substantial amount of trade between industrialized countries is intraindustry or intrafirm trade rather than interindustry trade.

True

According to OLI theory, a firm might be unwilling to license its production to a foreign firm for fear that its technology may be stolen or its brand name harmed, which leads the firm to internalize control over its asset and set up its own foreign subsidiary.

True

An increase in domestic demand for a product protected by a quota results in an increase in producer surplus for domestic firms, while for a tariff it would result in more imports.

True

Both tariffs and quotas lead to a decrease in imports, a decrease in domestic consumption, and an increase in domestic production

True

Chinese exports of toys and footwear can be explained by factor endowments, while Chinese exports of telecommunications equipment and computers and accessories can be explained by product-cycle analysis.

True

Comparative advantage can change over time.

True

Comparative advantage cannot account for a significant portion of world trade.

True

If countries have similar factors of production and similar productivities, most of their trade is likely to be intraindustry.

True

Intellectual property rights protection is a critical issue for the pharmaceutical industry among others.

True

Intraindustry trade can lead to lower prices and job creation in both the exporting and the importing nation.

True

Mercantilists perceived trade as a zero sum game.

True

Most currency trades in London do not involve the British pound.

True

Most migrants move from developing to developed nations.

True

Most trade between Mexico and the United States is intrafirm

True

Nontariff barriers to trade are less transparent than tariffs.

True

Nontariff measures are generally much more difficult to eliminate than tariffs and quotas because they are embedded more deeply in national economic policies.

True

Offshoring became a concern in the 1980s when modern communication and transport technology made it possible for firms to relocate production abroad.

True

The O in OLI theory stands for ownership, and the asset owned can be tangible or intangible.

True

The nation as a whole is better off from trade as long as the gains from the winners exceed the losses from the losers.

True

The rules for respecting property rights as they relate to trade were negotiated during the Uruguay Round (1986-1994) and culminated in the Trade Related Aspects Intellectual Property Rights (TRIPS) agreement.

True

Trading French wine for California wine is an example of intraindustry trade.

True

When did intellectual property rights become part of trade agreements?

Uruguay Round

A flexible exchange rate system guarantees a country will not experience an exchange rate crisis

false

A weak U.S. dollar leads to a higher volume of U.S. imports

false

All the countries of the EU participate in the Schengen Agreement.

false

An increase in interest rates causes that nation to experience an outflow of financial capital and causes its currency to depreciate.

false

Contractionary fiscal policy attempts to shift aggregate demand to the right.

false

Each of the 15 members of the European Union that joined before May 2004 use the euro as their currency.

false

Exchange rate crises are only associated with fixed exchange rate systems.

false

Expansionary fiscal policy is likely to lead a nation's currency to depreciate.

false

If inflation in the rest of the world is lower than inflation in Brazil, Brazil's currency (the real) would tend to appreciate.

false

Imports tend to fall whenever a nation's currency appreciates because foreign products become more expensive to domestic consumers.

false

In Canada, there are no preferences given to Canadian tv networks and programming over U.S. networks and programming.

false

It is more certain how expansionary monetary policy will affect the current account than how expansionary fiscal policy will affect it.

false

Most cohesion funds in the EU budget go to the wealthier EU member nations.

false

Tax systems in developing countries tend to be efficient and reliable.

false

Taxes, savings, and imports tend to magnify the effect of any spending change in the economy; that is, if investment spending initially increases, then spending will grow even more as taxes, savings, and imports increase, so the economic growth will accelerate

false

The Single European Act was expected to create economic benefits by reducing the costs and risks of currency market transactions.

false

The recent trend internationally has been for the executive and legislative branches of elected governments to get more control over monetary policy, as has been the case in countries such as the United States

false

There is no possibility of further widening of the European Union.

false

Unlike a banking crisis, an exchange rate crisis rarely results in a deep recession.

false

What matters most to importers and exporters is the nominal exchange rate.

false

If interest rates rise, what will happen to the nation's exchange rate?

it will appreciate

A common response to stop a depreciation of a currency is to use contractionary monetary policy, which could lead to a recession.

true

A debt crisis may lead to a banking crisis.

true

A large and growing current account deficit can be an indicator of a potential crisis.

true

An exchange rate crisis may lead to a banking crisis and disintermediation.

true

Contractionary fiscal policy can lead to a depreciation of the nation's currency.

true

Current research suggests that countries that adopt a pegged exchange rate may be more vulnerable to an exchange rate crisis.

true

Disintermediation is a problem associated with a banking crisis.

true

Even though Mexico is a developing country, the NAFTA market is very rich.

true

Exchange rates and banking systems are often the variables through which the contagion effects of a crisis are spread from one country to another.

true

Expansionary monetary policy involves an increase in the money supply and a fall in interest rates, leading to a positive expansion in income.

true

Expansionary monetary policy is likely to lead to a depreciation of the nation's currency.

true

Few countries have a higher GDP per capita than the weighted average of Canada, Mexico and the U.S.

true

Financial capital is highly volatile, and technological advances have reinforced this volatility

true

For any given increase in spending that is not directly caused by an increase in income, the impact on equilibrium GDP is greater than the initial spending increase

true

Free trade agreements between the United States and Canada started with the auto industry in the year 1965.

true

If Mexicans increasingly lose confidence in their domestic financial markets and move their assets to other countries, the peso will depreciate.

true

If U.S. consumers increase their demand for foreign products and foreign travel, the U.S. dollar would tend to depreciate as more dollars are supplied to foreign exchange markets.

true

If the Japanese central bank sells yen and buys U.S. dollars, the U.S. dollar will appreciate.

true

Many developing countries make the government budget one of the primary tools of long-run industrial development, with the government owning and operating industries such as steel mills, airlines, and phone companies.

true

Most Mexican workers can increase their wages if they migrate to the United States, a demand-pull factor for migration.

true

Quotas usually lead to larger deadweight losses than tariffs.

true

Subsidiarity requires nations to give up some of their national sovereignty.

true

The European Union is the oldest, largest, and most ambitious integration agreement in the world today.

true

The financial crisis that started in 2007 was unusual because it started in an advanced economy, the United States.

true

The most common type of macroeconomic imbalance is overly expansionary fiscal policies that create large government budget deficits, often financed by a high growth rate of the money supply.

true

The roots of the European Union are in agreements within the coal and steel industries

true

The two main types of economies generating current account surpluses from 2000 to 2007 were Asian exporters and oil producers.

true


Related study sets

Virtue Theory and Aristotle's Moral Virtues, Golden Mean

View Set

Question-Answer Relationship (QAR)

View Set

Chapter 3 Health and Physical Activity in Our Society

View Set

Module 7 - Reading and Interpreting the Income Statement

View Set

Week 8 Smartbook: Immune System & Body Defense

View Set

HESI Dosage Calculations Practice Exam, Hesi Pharmacology Review

View Set

SW 2000 Ch.13 Quiz: Communities at Risk and Housing

View Set